LSAT and Law School Admissions Forum

Get expert LSAT preparation and law school admissions advice from PowerScore Test Preparation.

User avatar
 Dave Killoran
PowerScore Staff
  • PowerScore Staff
  • Posts: 5852
  • Joined: Mar 25, 2011
|
#27308
Complete Question Explanation
(The complete setup for this game can be found here: lsat/viewtopic.php?t=11515)

The correct answer choice is (A)

The conditions in the question stem specify that no two consecutive calls are live or taped. Thus, no LL blocks or TT blocks can appear in the L/T stack, and the result is that L and T must alternate in this stack (regardless of which is first). Let’s examine each template (illustrated in the setup discussion) and see which ones can meet these conditions:

  • Template #1: If the first and last calls are taped, then the conditions in the question stem are met. Thus, the order in Template #1 is viable.

    Template #2: Because L is first and T is third, there is no call that can be aired second that will not cause a violation of the conditions in the question stem. Thus, this template cannot produce an order that satisfies the question.

    Template #3: Because T is third and L is fifth, there is no call that can be aired fourth that will not cause a violation of the conditions in the question stem. Thus, this template cannot produce an order that satisfies the question.

    Template #4: Because L is first and T is third and L is fifth, there are no calls that can be aired second or fourth that will not cause a violation of the conditions in the question stem. Thus, this template cannot produce an order that satisfies the question.


Thus, the only template that can meet the conditions in the question stem is Template #1, and so only one order of the callers meets the conditions, and answer choice (A) is correct.

Get the most out of your LSAT Prep Plus subscription.

Analyze and track your performance with our Testing and Analytics Package.